Complex in Logarithm

Algebra Level 3

Given that function F ( x ) = i 2 i + 3 ln ( i 3 x 2 + 2 x + i i x 2 + 2 x + i 3 ) F(x) = i^{2i+3}\ln { \left( \dfrac { { i }^{ 3 }{ x }^{ 2 }+{ 2x }+{ i } }{ { i }{ x }^{ 2 }+{ 2x }+{ i }^{ 3 } } \right) } for x ( 0 , 1 ) x \in (0,1) and F ( 1 3 ) = π e α π β F \left(\sqrt { \frac { 1 }{ 3 } }\right) = \dfrac { \pi }{ { e }^{ \alpha \pi }{ \beta } } , find α + β \alpha +\beta .

Notation: i = 1 i=\sqrt {-1} denotes the imaginary unit .


The answer is 4.

This section requires Javascript.
You are seeing this because something didn't load right. We suggest you, (a) try refreshing the page, (b) enabling javascript if it is disabled on your browser and, finally, (c) loading the non-javascript version of this page . We're sorry about the hassle.

3 solutions

Steven Chase
Feb 27, 2018

Note: For this problem to have a unique solution, we need to assume the principal root of negative 1 ( e i π / 2 ) (e^{i \pi / 2})

Evaluate the argument of the logarithm:

2 1 / 3 + ( 2 / 3 ) i 2 1 / 3 ( 2 / 3 ) i × 1 = 2 1 / 3 + ( 2 / 3 ) i 2 1 / 3 ( 2 / 3 ) i × 2 1 / 3 + ( 2 / 3 ) i 2 1 / 3 + ( 2 / 3 ) i = 1 + 3 i 2 = e i π / 3 \large{\frac{2 \sqrt{1/3} + (2/3)i}{2 \sqrt{1/3} - (2/3)i} \times 1 \\ = \frac{2 \sqrt{1/3} + (2/3)i}{2 \sqrt{1/3} - (2/3)i} \times \frac{2 \sqrt{1/3} + (2/3)i}{2 \sqrt{1/3} + (2/3)i} \\ = \frac{1 + \sqrt{3} i}{2} \\ = e^{i \pi / 3} }

The function therefore simplifies to:

F = i 2 i + 3 π 3 i = π 3 i 2 i + 4 = π 3 e ( i π / 2 ) ( 2 i + 4 ) = π 3 e π e i 2 π = π 3 e π \large{F = i^{2 i + 3} \,\, \frac{\pi}{3} i \\ = \frac{\pi}{3} \, i^{2 i + 4} \\ = \frac{\pi}{3} e^{(i \pi / 2)(2 i + 4)} \\ = \frac{\pi}{3} e^{- \pi} \, e^{i 2 \pi} \\ = \frac{\pi}{3 \, e^{\pi}}}

Nice solution Sir!!!

A Former Brilliant Member - 3 years, 3 months ago

Log in to reply

Sir we can also calculate it by first simplifying the function

A Former Brilliant Member - 3 years, 3 months ago

Let us first simplify the the given function i.e.

Let G(x) = l n ( i 3 x 2 + 2 x + i i x 2 + 2 x + i 3 ) = l n ( 2 x + i ( 1 x 2 ) 2 x i ( 1 x 2 ) ) ln{ \left( \frac { { { i }^{ 3 } }{ x }^{ 2 }+{ 2x }+{ i } }{ { i }{ x }^{ 2 }+{ 2x }+{ i }^{ 3 } } \right) }\quad =\quad ln{ \left( \frac { { 2x }+{ i }\left( { 1 }-{ x }^{ 2 } \right) }{ { 2x }-{ i }\left( { 1 }-{ x }^{ 2 } \right) } \right) } (by taking i as common)

Thus the above expression can be defined as the complex number (z) with real part 2x and imaginary part ( 1 x 2 ) \left( { 1 }-{ x }^{ 2 } \right)

therefore let us assume that magnitude of (z) is r and principal argument θ \theta , therefore above expression can be written as

l n ( r ( cos θ + i sin θ ) r ( cos θ i sin θ ) ) ln{ \left( \frac { r\left( { \cos { \theta } }+{ i }{ \sin { \theta } } \right) }{ r\left( { \cos { \theta } }-{ i }{ \sin { \theta } } \right) } \right) }

thus changing into euler form we get

l n ( e i θ e i θ ) = l n ( e 2 i θ ) = 2 i θ ln{ \left( \frac { { e }^{ i\theta } }{ { e }^{ -i\theta } } \right) }=ln{ \left( { e }^{ 2i\theta } \right) }=\quad 2i\theta and on more simplification

we get θ = tan 1 ( 1 x 2 2 x ) = cot 1 ( 2 x 1 x 2 ) = π 2 tan 1 ( 2 x 1 x 2 ) = π 2 2 tan 1 x \theta =\tan ^{ -1 }{ \left( \frac { { 1 }-{ x }^{ 2 } }{ 2x } \right) } =\cot ^{ -1 }{ \left( \frac { 2x }{ { 1 }-{ x }^{ 2 } } \right) } =\frac { \pi }{ 2 } -\tan ^{ -1 }{ \left( \frac { 2x }{ { 1 }-{ x }^{ 2 } } \right) } =\frac { \pi }{ 2 } -2\tan ^{ -1 }{ x } since x ϵ ( 0 , 1 ) \epsilon \left( 0,1 \right)

therefore G(x) = 2 i θ = i ( π 4 tan 1 x ) 2i\theta =i(\pi -4\tan ^{ -1 }{ x) }

Let H(x) = i 2 i + 3 { i }^{ 2i+3 }

therefore on simplification we get

i 2 i + 3 = ( i ) i 2 i = ( i ) ( e i π / 2 ) 2 i = ( i ) ( e π ) { i }^{ 2i+3 }=\left( -i \right) { i }^{ 2i }=\left( -i \right) { \left( { e }^{ i\pi /2 } \right) }^{ 2i }=\left( -i \right) \left( { e }^{ -\pi } \right)

Combining H(x) & G(x) we get

F(x) = ( e π ) ( π 4 tan 1 x ) \left( { e }^{ -\pi } \right) \left( { \pi }-{ 4\tan ^{ -1 }{ x } } \right)

therefore now required question can be solved

Chew-Seong Cheong
Feb 28, 2018

Solution inspired by @Ayush Mishra

F ( x ) = i 2 i + 3 ln ( i 3 x 2 + 2 x + i i x 2 + 2 x + i 3 ) Note that i 3 = ( i 2 ) i = i = i 2 i ( i ) ln ( i x 2 + 2 x + i i x 2 + 2 x i ) By Euler’s formula e i θ = cos θ + i sin θ = e i π 2 × 2 i ( i ) ln ( 2 x + i ( 1 x 2 ) 2 x i ( 1 x 2 ) ) = i e π ln ( 2 x 1 + x 2 + i 1 x 2 1 + x 2 2 x 1 + x 2 i 1 x 2 1 + x 2 ) By half-angle tangent substitution = i e π ln ( sin θ + i cos θ sin θ i cos θ ) where x = tan θ 2 = i e π ln ( cos ( π 2 θ ) + i sin ( π 2 θ ) cos ( π 2 θ ) + i sin ( π 2 θ ) ) = i e π ln ( e i ( π 2 θ ) e i ( π 2 θ ) ) By Euler’s formula e i ϕ = cos ϕ + i sin ϕ = i e π ln ( e 2 i ( π 2 θ ) ) = i e π ( 2 i ) ( π 2 θ ) Since x = tan θ 2 θ = 2 tan 1 x = π 2 2 tan 1 x e π \begin{aligned} F(x) & = i^{2i+\color{#3D99F6}3} \ln \left(\frac {{\color{#3D99F6}i^3}x^2+2x+i}{ix^2+2x+{\color{#3D99F6}i^3}}\right) & \small \color{#3D99F6} \text{Note that } i^3 = (i^2)i = -i \\ & = {\color{#D61F06}i}^{2i}{\color{#3D99F6}(-i)} \ln \left(\frac {{\color{#3D99F6}-i}x^2+2x+i}{ix^2+2x\color{#3D99F6}-i}\right) & \small \color{#D61F06} \text{By Euler's formula }e^{i\theta} = \cos \theta + i \sin \theta \\ & = {\color{#D61F06}e}^{{\color{#D61F06}i\frac \pi 2}\times 2i}(-i) \ln \left(\frac {2x+i(1-x^2)}{2x-i(1-x^2)}\right) \\ & = -ie^{-\pi} \ln \left( {\color{#3D99F6} \frac {\frac {2x}{1+x^2}+i\frac {1-x^2}{1+x^2}}{\frac {2x}{1+x^2}-i\frac {1-x^2}{1+x^2}}}\right) & \small \color{#3D99F6} \text{By half-angle tangent substitution} \\ & = -ie^{-\pi} \ln \left( {\color{#3D99F6} \frac {\sin \theta +i\cos \theta}{\sin \theta -i\cos \theta}}\right) & \small \color{#3D99F6} \text{where }x = \tan \frac \theta 2 \\ & = -ie^{-\pi} \ln \left( {\color{#3D99F6} \frac {\cos \left(\frac \pi 2 - \theta\right) +i\sin \left(\frac \pi 2 - \theta\right)}{\cos \left(\frac \pi 2 - \theta\right) +i\sin \left(\frac \pi 2 - \theta\right)}} \right) \\ & = -ie^{-\pi} \ln \left( {\color{#3D99F6} \frac {e^{i\left(\frac \pi 2 - \theta\right)}}{e^{-i\left(\frac \pi 2 - \theta\right)}}}\right) & \small \color{#3D99F6} \text{By Euler's formula }e^{i\phi} = \cos \phi + i \sin \phi \\ & = -ie^{-\pi} \ln \left(e^{2i\left(\frac \pi 2 - \theta\right)}\right) \\ & = -ie^{-\pi}(2i)\left(\frac \pi 2 - {\color{#3D99F6}\theta}\right) & \small \color{#3D99F6} \text{Since }x = \tan \frac \theta 2 \implies \theta = 2 \tan^{-1} x \\ & = \frac {\pi - 2\cdot{\color{#3D99F6}2\tan^{-1}x}}{e^\pi} \end{aligned}

F ( 1 3 ) = π 4 tan 1 ( 1 3 ) e π = π 4 π 6 e π = π 3 e π \implies F\left(\sqrt{\frac 13}\right) = \dfrac {\pi - 4\tan^{-1}\left(\sqrt{\frac 13}\right)}{e^\pi} = \dfrac {\pi - 4\cdot \frac \pi 6}{e^\pi} = \dfrac \pi{3e^\pi} . Therefore, α + β = 1 + 3 = 4 \alpha + \beta = 1+3 = \boxed{4} .


Previous solution

F ( x ) = i 2 i + 3 ln ( i 3 x 2 + 2 x + i i x 2 + 2 x + i 3 ) Note that i 3 = ( i 2 ) i = i = i 2 i ( i ) ln ( i x 2 + 2 x + i i x 2 + 2 x i ) By Euler’s formula e i θ = cos θ + i sin θ F ( 1 3 ) = e i π 2 × 2 i ( i ) ln ( i 1 3 + 2 3 + i i 1 3 + 2 3 i ) = e π ( i ) ln ( 2 3 + i 2 3 2 3 i 2 3 ) = i e π ln ( 4 3 ( 3 2 + i 1 2 ) 4 3 ( 3 2 i 1 2 ) ) By Euler’s formula again = i e π ln ( e π 6 i e π 6 i ) = i e π ln ( e π 3 i ) = i e π × π 3 i = π 3 e π \begin{aligned} F(x) & = i^{2i+\color{#3D99F6}3} \ln \left(\frac {{\color{#3D99F6}i^3}x^2+2x+i}{ix^2+2x+{\color{#3D99F6}i^3}}\right) & \small \color{#3D99F6} \text{Note that } i^3 = (i^2)i = -i \\ & = {\color{#D61F06}i}^{2i}{\color{#3D99F6}(-i)} \ln \left(\frac {{\color{#3D99F6}-i}x^2+2x+i}{ix^2+2x\color{#3D99F6}-i}\right) & \small \color{#D61F06} \text{By Euler's formula }e^{i\theta} = \cos \theta + i \sin \theta \\ F\left(\sqrt{\frac 13}\right) & = {\color{#D61F06}e}^{{\color{#D61F06}i\frac \pi 2}\times 2i}(-i) \ln \left(\frac {-i\frac 13 + \frac 2{\sqrt 3}+i}{i\frac 13+\frac 2{\sqrt 3}-i}\right) \\ & = e^{-\pi}(-i) \ln \left(\frac {\frac 2{\sqrt 3}+i\frac 23}{\frac 2{\sqrt 3}-i\frac 23}\right) \\ & = -i e^{-\pi}\ln \left(\frac {\frac 43 \color{#D61F06}\left(\frac {\sqrt 3}2+i\frac 12\right)}{\frac 43\color{#D61F06} \left(\frac {\sqrt 3}2-i\frac 12\right)} \right) & \small \color{#D61F06} \text{By Euler's formula again} \\ & = -i e^{-\pi}\ln \left(\frac {\color{#D61F06}e^{\frac \pi 6 i}}{\color{#D61F06}e^{-\frac \pi 6 i}} \right) \\ & = -i e^{-\pi}\ln \left(e^{\frac \pi 3 i} \right) \\ & = -i e^{-\pi}\times \frac \pi 3 i = \frac {\pi}{3e^\pi} \end{aligned}

Therefore, α + β = 1 + 3 = 4 \alpha + \beta = 1+3 = \boxed{4} .

Sir I had also written the solution, is it correct

A Former Brilliant Member - 3 years, 3 months ago

Log in to reply

Sorry, your solution is difficult to check.

Chew-Seong Cheong - 3 years, 3 months ago

0 pending reports

×

Problem Loading...

Note Loading...

Set Loading...